Đến nội dung

Hình ảnh

Marathon số học Olympic

- - - - -

  • Please log in to reply
Chủ đề này có 191 trả lời

#61
Zaraki

Zaraki

    PQT

  • Phó Quản lý Toán Cao cấp
  • 4273 Bài viết

Bài 23: (Sưu tầm)

Cho $n$ là một số nguyên dương. Gọi $a_n$ là số các số tự nhiên $k$ sao cho $\binom{n}{k}\equiv 1 \pmod{3}$ và $b_n$ là số các số tự nhiên $k$ sao cho $\binom{n}{k}\equiv 2 \pmod{3}$.

Chứng minh rằng: $a_n-b_n$ là một luỹ thừa của 2.

Lời giải. Ta biểu diễn $n$ và $k$ dưới dạng hệ cơ số $3$ như sau $$\begin{array}{l} n=x_m3^m+x_{m-1}3^{m-1}+ \cdots + x_1\cdot 3+x_0, \; x_j \in \{0,1,2 \} \\ k=y_m3^m+y_{m-1}3^{m-1}+ \cdots + y_1 \cdot 3+y_0, \; y_j \in \{ 0,1,2 \}. \end{array}$$

Áp dụng định lý Lucas ta có $$\binom{n}{k} \equiv 1 \pmod{3} \Leftrightarrow \prod_{j=0}^m \binom{x_j}{y_j} \equiv 1 \pmod{3}. \qquad (1)$$

Ta sẽ đi tìm số $k \le n$ thỏa mãn $\binom{n}{k} \equiv 1 \pmod{3}$. Để ý rằng mỗi bộ $m+1$ số $(y_0,y_1, \cdots y_m)$ sẽ cho ta một số $k$ khác nhau, do đó ta sẽ đi đếm bộ $(y_0,y_1, \cdots , y_m)$ thỏa mãn $\prod_{j=0}^m \binom{x_j}{y_j} \equiv 1 \pmod{3}$. Hiển nhiên rằng $x_j \ge y_j \; \forall j=\overline{0,m}$ vì nếu tồn tại $x_k<y_k$ thì $\binom{x_k}{y_k}=0$, không thể thỏa mãn điều kiện $(1)$.

Nếu $x_j=0$ thì $y_j=0$.

Nếu $x_j=1$ thì $y_j=0$ hoặc $y_j=1$.

Nếu $x_j=2$ thì $y_j=0$ hoặc $y_j=1$ hoặc $y_j=2$.

Ta thấy trường hợp $x_j=2,y_j=1$ thì $\binom{2}{1}=2$ còn tất cả các trường hợp còn lại đều cho $\binom{x_j}{y_j}=1$. Do đó để thỏa mãn $(1)$ thì phải có chẵn số bộ $(x_j,y_j)=(2,1)$. Ta đếm như sau:

Gỉa sử $n$ có $l_1$ số $x_j=0$, $l_2$ số $x_j=1$, $l_3$ số $x_j=2$. ($l_1+l_2+l_2=m+1$).

Có $l_1$ số $x_j=0$ nên sẽ có $1$ cách chọn cho $l_1$ số $y_j$ tương ứng (đều bằng $0$). Có $l_2$ số $x_j=1$ nên sẽ có $2^{l_2}$ cách chọn số $y_j$ tương ứng (bằng $0$ hoặc $1$).

Có $l_3$ số $x_j=2$ nên số cách chọn $y_j$ tương ứng sẽ là $\sum_{i=0}^{ \left \lfloor l_3/2 \right \rfloor} \binom{l_3}{2i}2^{l_3-2i}$.

Như vậy có $2^{l_2}\sum_{i=0}^{ \left \lfloor l_3/2 \right \rfloor} \binom{l_3}{2i}2^{l_3-2i}$ cách chọn ra số $k \le n$ thỏa mãn $\binom nk \equiv 1 \pmod{3}$, hay nói cách khác $a_n= 2^{l_2} \sum_{i=0}^{ \left \lfloor l_3/2 \right \rfloor} \binom{l_3}{2i}2^{l_3-2i}$.

 

Hoàn toàn tương tự, ta cũng tính được $b_n=2^{l_2} \sum_{i=0}^{ \left \lfloor l_3/2 \right \rfloor} \binom{l_3}{2i+1}2^{l_3-2i-1}$. Do đó $$a_n-b_n= 2^{l_2} \left(  \sum_{i=0}^{ \left \lfloor l_3/2 \right \rfloor} \binom{l_3}{2i}2^{l_3-2i} - \sum_{i=0}^{ \left \lfloor l_3/2 \right \rfloor} \binom{l_3}{2i+1}2^{l_3-2i-1} \right)=2^{l_2} \cdot (2-1)^{l_3}= \boxed{2^{l_2}}.$$

Bài toán được chứng minh. $\blacksquare$


Discovery is a child’s privilege. I mean the small child, the child who is not afraid to be wrong, to look silly, to not be serious, and to act differently from everyone else. He is also not afraid that the things he is interested in are in bad taste or turn out to be different from his expectations, from what they should be, or rather he is not afraid of what they actually are. He ignores the silent and flawless consensus that is part of the air we breathe – the consensus of all the people who are, or are reputed to be, reasonable.

 

Grothendieck, Récoltes et Semailles (“Crops and Seeds”). 


#62
Zaraki

Zaraki

    PQT

  • Phó Quản lý Toán Cao cấp
  • 4273 Bài viết

Hi

Mình nghĩ bài toán sẽ khó hơn nếu $a+b+c+d \mid a^n+b^n+c^n+d^n$ chỉ đúng với $n=2,3$. Lời giải của mình chỉ sử dụng hai cái này.

 

Ps: Điểm... :D


Discovery is a child’s privilege. I mean the small child, the child who is not afraid to be wrong, to look silly, to not be serious, and to act differently from everyone else. He is also not afraid that the things he is interested in are in bad taste or turn out to be different from his expectations, from what they should be, or rather he is not afraid of what they actually are. He ignores the silent and flawless consensus that is part of the air we breathe – the consensus of all the people who are, or are reputed to be, reasonable.

 

Grothendieck, Récoltes et Semailles (“Crops and Seeds”). 


#63
Ego

Ego

    Thượng sĩ

  • Điều hành viên OLYMPIC
  • 296 Bài viết

Zaraki

Hiện Zaraki đang dẫn đầu lẫn dẫn cuối (=))) danh sách với $5$ điểm, các bạn thi đua nào!
Điểm số sẽ được cập nhật thường xuyên ở #2.
 

 

 

$$\begin{array}{|c|c|c|} \hline \textbf{STT}& \textbf{Tên trên diễn đàn} &  \textbf{Điểm}   \\ \hline
 1 & \text{bangbang1412} & 4 \\ \hline
2  & \text{baopbc} & 0.5 \\ \hline
3  & \text{canhhoang30011999} & 2 \\ \hline
4  & \text{Ego} & 3.5 \\ \hline
5  & \text{hoanglong2k} & 1 \\ \hline
6  & \text{Hoang Nhat Tuan}& 1 \\ \hline
7  & \text{I Love MC} & 2.5 \\ \hline
8  & \text{IMOer} & 4 \\ \hline
9  & \text{ngocanh99} & 1 \\ \hline
10  & \text{Visitor} & 1 \\ \hline
11  & \text{Zaraki} & 5\\ \hline
\end{array}$$


Các bạn nào có phàn nàn gì về spam trong topic hay là điểm số cứ PM mình. Xin cám ơn.

Bài viết đã được chỉnh sửa nội dung bởi Ego: 30-05-2016 - 13:59


#64
Long Phi

Long Phi

    Binh nhất

  • Thành viên
  • 26 Bài viết

Bài 24:Phương trình tương đương với:

$(4x+y)^{2}-5y^{2}=4n(*)$

Phương trình vô số nghiệm $<=>$ phương trình $a^{2}-5b^{2}=4n(**)$ có vô số nghiệm thỏa $a-b$ chia hết cho 4

Dãy nghiệm của (**) là

$x’ = ax  -5by$

$y’ = ay + bx$

với $a, b$ là nghiệm cơ sở của $x^{2}-5y^{2}=1$

Nếu $b$ lẻ thì VT đồng dư $3$ hoặc $5$ mod $8$ $=>$ b chẵn

Do (*) có nghiệm nên $x-y$ chia hết cho 4

$=> x'-y' \equiv a(x-y)+b(x-y) \equiv 0(mod4)$


Bài viết đã được chỉnh sửa nội dung bởi Long Phi: 30-05-2016 - 14:23


#65
IMOer

IMOer

    Hạ sĩ

  • Thành viên
  • 50 Bài viết

Lời giải bài 24:

Với $n=0$ thì phương trình ${{x}^{2}}+xy-{{y}^{2}}=0$ có nghiệm duy nhất $\left( x,y \right)=\left( 0,0 \right)$ hay kết luận của bài toán sai.

Ta chỉ chứng minh với $n$ là số nguyên dương.

Nhận thấy $\left( x,y \right)$ là nghiệm của phương trình ${{x}^{2}}+xy-{{y}^{2}}=n$ thì $\left( -x,-y \right)$ cũng là nghiệm của phương trình này.

Gọi $\left( {{x}_{0}},{{y}_{0}} \right)$ là nghiệm của phương trình ${{x}^{2}}+xy-{{y}^{2}}=n$ sao cho $x_0>0$ hoặc $y_0>0$.

Xét dãy: $\left\{ \begin{array}{l}{{x}_{k}}={{F}_{k}}{{x}_{0}}+{{F}_{k+1}}{{y}_{0}}\\{{y}_{k}}={{F}_{k+1}}{{x}_{0}}+{{F}_{k+2}}{{y}_{0}}\end{array} \right.$ với ${{F}_{k}}$ là số hạng thứ $k$ của dãy Fibonacci.

Khi đó ta có: $\left( {{x}_{k}},\ {{y}_{k}} \right)$ cũng là nghiệm của phương trình ${{x}^{2}}+xy-{{y}^{2}}=n$.

Từ đó suy ra phương trình ${{x}^{2}}+xy-{{y}^{2}}=n$ có vô số nghiệm nguyên.



#66
canhhoang30011999

canhhoang30011999

    Thiếu úy

  • Thành viên
  • 634 Bài viết

Lời nhắn tới bạn canhhoang30111999

không thể có Th đó được vì mình đang xét th 

$p^{t}=(ax+by)^{2}+(ay-bx)^{2}=(ax-by)^{2}+(ay-bx)^{2}$ nên nếu chúng có ước nguyên tố chung thì phải là p



#67
IMOer

IMOer

    Hạ sĩ

  • Thành viên
  • 50 Bài viết

Lời giải bài 20*:

Vì $\left( a-b \right)\left( a-c \right)\left( a-d \right)\left( b-c \right)\left( b-d \right)\left( c-d \right)\ \vdots \ 12$ với mọi $a,b,c,d\in \mathbb{Z}$ nên $\gcd \left( a+b+c+d,12 \right)=1$.

Gọi $p$ là ước nguyên tố của $a+b+c+d$. Khi đó $p\ne 2;\ p\ne 3$.

Giả sử: $a+b$ không chia hết cho $p$.

Khi đó ta có: $a+b\equiv -\left( c+d \right)\ \left( \bmod \ p \right)\Rightarrow {{\left( a+b \right)}^{3}}\equiv -{{\left( c+d \right)}^{3}}\ \left( \bmod \ p \right)$

Mà ${{a}^{3}}+{{b}^{3}}\equiv -\left( {{c}^{3}}+{{d}^{3}} \right)\ \left( \bmod \ p \right)$ nên suy ra: $3ab\left( a+b \right)=-3cd\left( c+d \right)\ \left( \bmod \ p \right)$.

Do $a+b\not\equiv 0\left( \bmod \ p \right)$ và $p\ne3$ nên suy ra được: $ab\equiv cd\ \left( \bmod \ p \right)$.

Lại có: ${{\left( a+b \right)}^{2}}\equiv {{\left( c+d \right)}^{2}}\ \left( \bmod \ p \right)\Rightarrow {{a}^{2}}+{{b}^{2}}\equiv {{c}^{2}}+{{d}^{2}}\ \left( \bmod \ p \right)$

Mà ${{a}^{2}}+{{b}^{2}}+{{c}^{2}}+{{d}^{2}}\equiv 0\ \left( \bmod \ p \right)\Rightarrow {{a}^{2}}+{{b}^{2}}\equiv {{c}^{2}}+{{d}^{2}}\equiv 0\ \left( \bmod \ p \right)$ do $p\ne 2$.

Vì $a,b,c,d$ không chia hết cho $p$ nên suy ra $p$ chỉ có dạng $4k+1$.

Nên suy ra tất cả các ước nguyên tố của $a+b+c+d$ đều có dạng $4k+1$, điều phải chứng minh.



#68
Ego

Ego

    Thượng sĩ

  • Điều hành viên OLYMPIC
  • 296 Bài viết

@IMOer: Lời giải bài 24 đẹp với lạ quá, bạn có thể giới thiệu cho mọi người tại sao bạn lại nghĩ đến việc chọn dãy như thế được không ạ?



#69
bangbang1412

bangbang1412

    Độc cô cầu bại

  • Phó Quản lý Toán Cao cấp
  • 1668 Bài viết

Tớ nghĩ cứ đưa nó về dãy truy hồi cấp $1$ rồi chọn thôi 


$$[\Psi_f(\mathbb{1}_{X_{\eta}}) ] = \sum_{\varnothing \neq J} (-1)^{\left|J \right|-1} [\mathrm{M}_{X_{\sigma},c}^{\vee}(\widetilde{D}_J^{\circ} \times_k \mathbf{G}_{m,k}^{\left|J \right|-1})] \in K_0(\mathbf{SH}_{\mathfrak{M},ct}(X_{\sigma})).$$


#70
IMOer

IMOer

    Hạ sĩ

  • Thành viên
  • 50 Bài viết

Mình xin phép đề xuất bài tiếp theo

Bài 25: (From a France competition)

Cho $p\geq 5$ là một số nguyên tố. Chứng minh tồn tại 2 số nguyên tố phân biệt $q,r$ bé hơn $p$ thoả mãn $q^{p-1}\not\equiv 1(\bmod{p^2})$ và $r^{p-1}\not\equiv 1(\bmod{p^2})$



#71
IMOer

IMOer

    Hạ sĩ

  • Thành viên
  • 50 Bài viết

@IMOer: Lời giải bài 24 đẹp với lạ quá, bạn có thể giới thiệu cho mọi người tại sao bạn lại nghĩ đến việc chọn dãy như thế được không ạ?

Mình xin chia sẻ với Ego và các bạn khác, tại sao mình lại chọn dãy như vậy.

Từ phương trình ban đầu ta viết lại được dưới dạng: $(2x+y)^2-5y^2=n$.

Đến đây, mình nghĩ tới phương trình Pell liên kết với phương trình này là $x^2-5y^2=1$.

Phương trình Pell này có nghiệm cơ bản $(x,y)=(9,4)$.

Từ đó mình tìm ra được là $(x,y)$ là nghiệm thì $(5x+8y,8x+13y)$ cũng là nghiệm.

Linh cảm mách bảo mình $5,8,13$ là 3 số liên tiếp của dãy Fibonacci nên khả năng 3 số hạng liên tiếp khác của dãy Fibonacci cũng đúng.

Và nó đúng thật.

Đến đây thì mọi người sẽ thấy lời giải đó hoàn toàn tự nhiên chứ không hề có mưu mẹo gì.



#72
Zaraki

Zaraki

    PQT

  • Phó Quản lý Toán Cao cấp
  • 4273 Bài viết

Mình xin phép đề xuất bài tiếp theo

Bài 25: (From a France competition)

Cho $p\geq 5$ là một số nguyên tố. Chứng minh tồn tại 2 số nguyên tố phân biệt $q,r$ bé hơn $p$ thoả mãn $q^{p-1}\not\equiv 1(\bmod{p^2})$ và $r^{p-1}\not\equiv 1(\bmod{p^2})$

Lời giải. Ta thấy rằng $(p-1)^{p-1} \equiv (p-1)p\cdot (-1)^{p-2}+(-1)^{p-1} \equiv p+1 \pmod{p^2}$. 

Nếu tất cả các ước nguyên tố $q$ của $p-1$ đều có $q^{p-1} \equiv 1 \pmod{p^2}$ thì $(p-1)^{p-1} \equiv 1 \pmod{p^2}$, suy ra mâu thuẫn. Vậy tồn tại ít nhất một ước nguyên tố $q$ của $p-1$ sao cho $q^{p-1} \not\equiv 1 \pmod{p^2}$.

 

Ta chứng minh tồn tại số $p-h$ sao cho $q \nmid p-h$ và $q \nmid h$. Thật vậy, do có $\left \lfloor \frac{p-1}{q} \right \rfloor$ số chia hết cho $q$ từ $1$ đến $p-1$ nên sẽ có ít nhất $p-1- \left \lfloor \frac{p-1}{q} \right \rfloor \ge \frac{p+1}{2}$ số không chia hết cho $q$. Nếu ta chia các số từ $1$ đến $p-1$ thành các cặp $(i,p-i)$ thì có tất cả $\frac{p-1}{2}$ cặp nên theo nguyên lí Dirichlet ta suy ra tồn tại $h$ thoả mãn $\gcd (q,h(p-h))=1$.

 

 Khi đó xét $(p-h)^{p-1} \equiv (p-1)p(-h)^{p-2}+(-h)^{p-1} \equiv h^{p-2}(p+h) \pmod{p^2}$.

 

TH1. Nếu $h^{p-1} \equiv 1 \pmod{p^2}$ thì ta suy ra $(p-h)^{p-1} \equiv h^{p-2}p \not\equiv 1 \pmod{p^2}$ suy ra tồn tại một ước nguyên tố $r$ của $p-h$ sao cho $r^{p-1} \not\equiv 1 \pmod{p^2}$. Hiển nhiên rằng $r \ne q$ vì $\gcd (p-h,q)=1$. 

 

TH2. Nếu $h^{p-1} \not\equiv 1 \pmod{p^2}$ thì ta suy ra tồn tại ước nguyên tố $r$ của $h$ sao cho $r^{p-1} \not\equiv 1 \pmod{p^2}$. Ta cũng thấy rằng $r \ne q$ vì $q \nmid h$.

 

Như vậy luôn tồn tại hai số nguyên tố phân biệt $q,r<p$ sao cho $q^{p-1} \not\equiv 1 \pmod{p^2}, r^{p-1} \not\equiv 1 \pmod{p^2}$.  $\blacksquare$

 

Bài 26. [AoPS] Cho $n,k$ là các số nguyên dương sao cho $2 \le k \le n$. Chứng minh rằng tồn tại $n$ só nguyên dương phân biệt sao cho tổng bất kì $k$ số trong chúng là ước của tích của chính $k$ số đó.


Bài viết đã được chỉnh sửa nội dung bởi Zaraki: 31-05-2016 - 02:33

Discovery is a child’s privilege. I mean the small child, the child who is not afraid to be wrong, to look silly, to not be serious, and to act differently from everyone else. He is also not afraid that the things he is interested in are in bad taste or turn out to be different from his expectations, from what they should be, or rather he is not afraid of what they actually are. He ignores the silent and flawless consensus that is part of the air we breathe – the consensus of all the people who are, or are reputed to be, reasonable.

 

Grothendieck, Récoltes et Semailles (“Crops and Seeds”). 


#73
bangbang1412

bangbang1412

    Độc cô cầu bại

  • Phó Quản lý Toán Cao cấp
  • 1668 Bài viết

Lời giải bài 26 : Xét bộ số nguyên dương $(a_{1},a_{2},...a_{n})$ và số $k$ như đề bài

Chọn số số như sau , chọn một bộ số $(b_{1},...b_{n})$ rồi đặt $S=\prod_{i=1}^{m} i^{b_{i}}$ và chọn $a_{i}=i.S$

Ta có $a_{i_{1}}+...+a_{i_{k}}=1^{b_{1}}2^{b_{2}}...(i_{1}+...i_{k})^{1+b_{i_{1}+..i_{k}}}...m^{b_{m}}$

Trong đó $m>1+2+..+n$

Nhưng nhận ra rằng tích các số này là $S^{n}.n!$ ta chỉ cần so sánh lũy thừa của $(i_{1}+...i_{k})$ trong tích này , hiển nhiên trong $S^{n}$ chứa số $i_{1}+...i_{k}$ ta cần chứng minh bdt sau

$1+b_{i_{1}+...i_{k}} \leq nb_{i_{1}+....i_{k}}$ 

Do đó ta có đpcm 

P/s : Latex của diễn đàn sao ấy nhỉ :) mình check mấy lần toàn auto sai

Nợ bài $27$ một lúc

Bài 27 : Cho $a,b>1$ là hai số nguyên dương thỏa mãn $a^{n}-1|b^{n}-1$ với mọi $n$ nguyen dương . CMR $b$ là một lũy thừa của $a$


Bài viết đã được chỉnh sửa nội dung bởi bangbang1412: 30-05-2016 - 20:00

$$[\Psi_f(\mathbb{1}_{X_{\eta}}) ] = \sum_{\varnothing \neq J} (-1)^{\left|J \right|-1} [\mathrm{M}_{X_{\sigma},c}^{\vee}(\widetilde{D}_J^{\circ} \times_k \mathbf{G}_{m,k}^{\left|J \right|-1})] \in K_0(\mathbf{SH}_{\mathfrak{M},ct}(X_{\sigma})).$$


#74
Zaraki

Zaraki

    PQT

  • Phó Quản lý Toán Cao cấp
  • 4273 Bài viết

Một cách chọn khác cho bài 26. Xét $n$ số $A,2A,3A, \cdots , nA$, Xét $b_1,b_2, \cdots, b_l$ là tổng của $k$ số bất kì từ $1$ đến $n$ với $l=\tbinom nk$. Ta chọn $A=\prod_{i=1}^l b_i$ là xong. Thật vậy, xét bất kì $k$ số $c_1A,c_2A, \cdots , c_k A \; (1 \le c_i \le n, c_i \ne c_j)$ thì $$\sum_{i=1}^kc_iA=Ab_h \mid A^2 \mid \prod_{i=1}^k (c_iA).$$


Discovery is a child’s privilege. I mean the small child, the child who is not afraid to be wrong, to look silly, to not be serious, and to act differently from everyone else. He is also not afraid that the things he is interested in are in bad taste or turn out to be different from his expectations, from what they should be, or rather he is not afraid of what they actually are. He ignores the silent and flawless consensus that is part of the air we breathe – the consensus of all the people who are, or are reputed to be, reasonable.

 

Grothendieck, Récoltes et Semailles (“Crops and Seeds”). 


#75
CandyPanda

CandyPanda

    Hạ sĩ

  • Thành viên
  • 64 Bài viết

Bài 27: Ta sẽ chứng minh $b\vdots a$, khi đó đặt $b=ac$ thì bài toán quy về $a^{n}-1|c^{n}-1$

 

Xét dãy: $x_{n,1}=\frac{b^{n}-1}{a^{n}-1}$, $x_{n,k+1}=bx_{n,k}-a^{k}x_{n,k}$

Theo giả thiết thì $x_{n,k}$ luôn nguyên.

 

Ta sẽ chứng minh tồn tại k sao cho $lim x_{n,k}$ tiến tới $0$ khi $n$ tiến tới vô hạn (1)

 

Thật vậy:

Ta có nhận xét sau: $x_{n,k}=\frac{cb^{n}+c_{k-1}a^{(k-1)n}+c_{k-2}a^{(k-2)n}+...+c_{1}a^{n}+c_{0}}{(a^{n+k-1}-1)(a^{n+k-2}-1)...(a^{n}-1)}$

Chứng minh theo quy nạp, với $c,c_{0},c_{1},...c_{k-1}$ là các số cố định nhưng thay đổi theo bộ (n,k).

Nói cách khác là khi phân tích thì $x_{n,k}$ thì các số trên tử số chỉ có thể là $b^{n},a^{(k-1)n}, a^{(k-2)n},...,a^{n}$ và hệ số tự do.

Từ đây ta có, chọn k sao cho $a^{n+k-1}.a^{n+k-2}...a^{n} > b^{n}, a^{(k-1)n}$ thì $lim x_{n,k}=0$, dễ dàng chọn được $k $sao cho $a^{k} > b$ là được

Tóm lại (1) xong.

 

Từ (1)

Khi đó, tồn tại j nhỏ nhất sao cho $lim x_{n,j}$ (hiển nhiên $j>1$), tức là tồn tại $N$ đủ lớn sao cho từ$ N$ trở đi thì $bx_{n,j-1}=a^{j}x_{n+1,j-1}$

Tương đương với $x_{n,j-1}=(\frac{b}{a^{j}})^{n-N}x_{N,j-1}$. Nếu $b$ không là lũy thừa của a, mà $x_{n,k}$ luôn nguyên nên $x_{N,j-1}=0$, hay $x_{n,j-1}=0$ từ N trở đi, trái với cách chọn j ở trên, từ đó dẫn tới giả sử vô lý, hay $b$ chia hết cho $a$ => đpcm

Đề xuất bài 28 :  Kí hiệu $\pi(n)$ là số các số nguyên tố không vượt quá $n$ . CMR có vô số số $n$ mà $n$ là bội của $\pi(n)$


Bài viết đã được chỉnh sửa nội dung bởi bangbang1412: 30-05-2016 - 22:30


#76
bangbang1412

bangbang1412

    Độc cô cầu bại

  • Phó Quản lý Toán Cao cấp
  • 1668 Bài viết

Lời giải bài 28 : 

Bài toán quy về chứng minh $lim \frac{\pi(n)}{n}=0$ thật vậy ta có chặn sau $\frac{1}{6logn} < \frac{\pi(n)}{n} < \frac{6}{logn}$ nên ta có ngay đpcm

:( Lại dùng đao to bổ gà rồi . Ngoài ra để chứng minh cái $lim$ trên là $0$ có thể dùng tích $n$ số nguyên tố đầu tiên không vượt quá $4^{n}$ 

Ngoài ra còn chứng minh với mọi $k$ thì có $n$ mà $n = k\pi(n)$ ( cái này không rõ mọi người thử cm xem )

Bài 29 : Cho nó cùng chủ đề . Chứng minh không tồn tại hai đa thức $P(x),Q(x)$ mà $\pi(x)=\frac{P(x)}{Q(x)}$


Bài viết đã được chỉnh sửa nội dung bởi bangbang1412: 30-05-2016 - 23:49

$$[\Psi_f(\mathbb{1}_{X_{\eta}}) ] = \sum_{\varnothing \neq J} (-1)^{\left|J \right|-1} [\mathrm{M}_{X_{\sigma},c}^{\vee}(\widetilde{D}_J^{\circ} \times_k \mathbf{G}_{m,k}^{\left|J \right|-1})] \in K_0(\mathbf{SH}_{\mathfrak{M},ct}(X_{\sigma})).$$


#77
Visitor

Visitor

    Hạ sĩ

  • Thành viên
  • 66 Bài viết

Lời giải bài 29.

Giả sử tồn tại $P(x),Q(x)$

Do $lim \pi(x)=+\propto$ nên $degP>degQ$

Mà $lim \frac{\pi(x)}{x}=0\Rightarrow lim\frac{P(x)}{xQ(x)}=0\Rightarrow degP<degQ+1$
Mâu thuẫn. Vậy ta có đpcm.

Bài 30. Chứng minh rằng tồn tại vô số số tự nhiên $n$ sao cho tất cả ước nguyên tố của $n^2+n+1$ đều ko lớn hơn $\sqrt{n}$


Bài viết đã được chỉnh sửa nội dung bởi Visitor: 31-05-2016 - 09:10

__________

Bruno Mars


#78
I Love MC

I Love MC

    Đại úy

  • Thành viên nổi bật 2016
  • 1861 Bài viết

Lời giải bài $30$ 
Chọn $n=16.a^{8}$ 
$n^2+n+1=(4a^4-2a^2+1)(4a^4+2a^2+1)(4a^4-4a^3+2a^2-2a+1)(4a^4+4a^3+2a^2+2a+1)$

Ta chỉ cần quan tâm đến việc chọn $4a^4+4a^3+2a^2+2a+1,4a^4+2a^2+1$ như thế nào là đúng đắn nhất 
Chọn $a \equiv 1 \pmod{7},a \equiv 1 \pmod{13}$ hay $a \equiv 1 \pmod{91}$ 
Khi đó $\sqrt{n}=4a^4>\frac{4a^4+4a^3+2a^2+2a+1}{13},\frac{4a^4+2a^2+1}{7}$ 
Như vậy các ước nguyên tố sau luôn sẽ bé hơn mấy số này 
P/s : Khá khó khăn trong việc chọn modulo 
Bài 31 : Với mỗi số nguyên $a$ , hãy tính số nghiệm $(x,y,z)$ của phương trình đồng dư $x^2+y^2+z^2 \equiv 2axyz \pmod{p}$ 


Bài viết đã được chỉnh sửa nội dung bởi baopbc: 31-05-2016 - 09:57


#79
Visitor

Visitor

    Hạ sĩ

  • Thành viên
  • 66 Bài viết

Lời giải bài $30$ 
Chọn $n=16.a^{8}$ 
$n^2+n+1=(4a^4-2a^2+1)(4a^4+2a^2+1)(4a^4-4a^3+2a^2-2a+1)(4a^4+4a^3+2a^2+2a+1)$

Ta chỉ cần quan tâm đến việc chọn $4a^4+4a^3+2a^2+2a+1,4a^4+2a^2+1$ như thế nào là đúng đắn nhất 
Chọn $a \equiv 1 \pmod{7},a \equiv 1 \pmod{13}$ hay $a \equiv 1 \pmod{91}$ 
Khi đó $\sqrt{n}=4a^4>\frac{4a^4+4a^3+2a^2+2a+1}{13},\frac{4a^4+2a^2+1}{7}$ 
Như vậy các ước nguyên tố sau luôn sẽ bé hơn mấy số này 
P/s : Khá khó khăn trong việc chọn modulo 

Nếu đã phân tích được thế kia thì chọn modulo có khó khăn gì đâu em. Cho anh hỏi em biến đổi biểu thức kia ntn thế?   quá khủng


Bài viết đã được chỉnh sửa nội dung bởi Visitor: 31-05-2016 - 10:45

__________

Bruno Mars


#80
IMOer

IMOer

    Hạ sĩ

  • Thành viên
  • 50 Bài viết

Lời giải bài 31:

Xét các số trên $\mathbb{Z}/\mathbb{Z}_p$.

Ta có: $x^2+y^2+z^2\equiv 2axyz\ (\bmod{p}) \Leftrightarrow (z-axy)^2\equiv (a^2x^2-1)y^2-x^2\ (\bmod{p})$.

Với mỗi cặp số $x,y\in\{0,1,\ldots,p-1\}$ thì số nghiệm $z$ là: $1+\left(\dfrac{(a^2x^2-1)y^2-x^2}{p}\right)$.

Suy ra số nghiệm của phương trình đồng dư đã cho là:

\[S=p^2+\sum_{x=0}^{p-1}\sum_{y=0}^{p-1}\left(\dfrac{(a^2x^2-1)y^2-x^2}{p}\right)\]

Áp dụng bổ đề: $\sum_{x=0}^{p-1} \left(\dfrac{ax^2+bx+c}{p}\right)=\left\{\begin{array}{l}-\left(\dfrac{a}{p}\right)&,p\nmid b^2-4ac\\(p-1)\left(\dfrac{a}{p}\right)&,p|b^2-4ac\end{array}\right.$ ta có:

\[\sum_{y=0}^{p-1} \left(\dfrac{(a^2x^2-1)y^2-x^2}{p}\right)=\left\{\begin{array}{l}-\left(\dfrac{a^2x^2-1}{p}\right)&,ax\not\equiv \pm1\ (\bmod{p})\\(p-1)\left(\dfrac{a^2x^2-1}{p}\right)&,ax\equiv \pm1\ (\bmod{p})\end{array}\right.\]

Suy ra: \[S=p^2+2p\left(\dfrac{-1}{p}\right)-\sum_{x=0}^{p-1}\left(\dfrac{a^2x^2-1}{p}\right)=\left(p+\left(\dfrac{-1}{p}\right)\right)^2\]

 

Bài 32: (Nguồn: Sưu tầm)

Tìm tất cả các số nguyên dương $m\equiv 2\ (\bmod{4})$ sao cho tồn tại các số tự nhiên $n, k$ và số nguyên tố $p$ sao cho $\dfrac{m^{2^n-1}-1}{m-1}=m^n+p^k$.






1 người đang xem chủ đề

0 thành viên, 1 khách, 0 thành viên ẩn danh